0% found this document useful (0 votes)
18 views6 pages

Problem Set 5 - Solutions

fm212

Uploaded by

1253853533
Copyright
© © All Rights Reserved
We take content rights seriously. If you suspect this is your content, claim it here.
Available Formats
Download as PDF, TXT or read online on Scribd
0% found this document useful (0 votes)
18 views6 pages

Problem Set 5 - Solutions

fm212

Uploaded by

1253853533
Copyright
© © All Rights Reserved
We take content rights seriously. If you suspect this is your content, claim it here.
Available Formats
Download as PDF, TXT or read online on Scribd
You are on page 1/ 6

FM212 - Principles of Finance

Michaelmas Term: Asset Pricing

Problem Set 5 - Risk, Return and the Cost of Capital

1. Answer the following questions:

(a) What is the correlation coefficient between two stocks that gives the maximum reduction
in risk (variance) for a two-stock portfolio (assuming that the portfolio contains long
positions in both stocks)?
(b) Historical nominal annual returns for stock A are -8%, +10% and +22%. The nominal
returns for the market portfolio in the same years are +6%, +18% and +24%. Calculate
the beta for stock A.
(c) The correlation coefficient between stock B and the market portfolio is 0.8. The standard
deviation of stock B is 35% and that of the market is 20%. Calculate the beta of the
stock.

Solution (a): To answer this question we need to know how to calculate the risk (variance)
of a 2 asset portfolio. The formula for the variance of a 2 asset portfolio can be written as:

σR2 p = wA
2 2 2 2
σRA + wB σRB + 2wA wB Cov (RA , RB )

It is preferable to rewrite the formula in terms of the correlation coefficient because the cor-
relation coefficient is bounded within the range +1 to -1. Given the relationship between
covariance and the correlation coefficient we can rewrite the formula as:

Cov (RA , RB ) = σRA σRB ρRA ,RB

σR2 p = wA
2 2 2 2
σRA + wB σRB + 2wA wB σRA σRB ρRA ,RB

1
If we hold long positions in both stocks the correlation coefficient that gives the maximum
reduction in risk for a two-stock portfolio is -1.

If one stock is sold short and another stock is a long position in the portfolio then a correla-
tion of +1 is actually best to minimize portfolio risk.

Solution (b): The mean return on Stock A is:

−0.08 + 0.1 + 0.22


R̄A = = 0.08
3

The mean return on the market is:

0.06 + 0.18 + 0.24


R̄M = = 0.16
3

The covariance between the returns on Stock A and the returns on the market is:

(−0.08 − 0.08) (0.06 − 0.16) + (0.1 − 0.08) (0.18 − 0.16) + (0.22 − 0.08) (0.24 − 0.16)
= 0.0138
3−1

Cov (RA , RM ) = 0.0138

The variance of the returns on the market is:

(0.06 − 0.16)2 + (0.18 − 0.16)2 + (0.24 − 0.16)2


σR2 M = = 0.0084
3−1

2
Therefore the beta of Stock A is:

Cov (RA , RM ) 0.0138


βA = 2
= = 1.643
σRM 0.0084

Solution (c): The beta of Stock B is:

Cov (RB , RM ) σR σR ρR ,R σR ρR ,R
βB = 2
= B M2 B M = B B M
σRM σRM σ RM

0.35 (0.8)
βB = = 1.4
0.2

2. You are considering how to invest part of your retirement savings. You have decided to put
£200,000 into three stocks: 50% of the money in stock A (currently £25 per share), 25% of
the money in stock B (currently £80 per share) and the remainder in stock C (currently £2
per share). If stock A goes up to £30 per share, stock B drops to £60 per share and stock C
rises to £3 per share:

(a) What is the new value of the portfolio?


(b) What return did the portfolio earn?
(c) If you dont buy or sell any shares after the price change, what are your new portfolio
weights?

Solution (a): We first need to calculate the number of shares held for Stocks A, B and C:

£200, 000 (0.5)


NA = = 4000 Shares
£25

3
£200, 000 (0.25)
NB = = 625 Shares
£80

£200, 000 (0.25)


NC = = 25, 000 Shares
£2

When the stock prices change the new value of the portfolio is:

£30 (4000) + £60 (625) + £3 (25, 000) = £232, 500

Solution (b): The return of the portfolio is:

£232, 500 − £200, 000


Rp = = 0.1625
£200, 000

Solution (c): The new portfolio weights are:

£30 (4000)
wA = = 0.5161
£232, 500

£60 (625)
wB = = 0.1613
£232, 500

£3 (25, 000)
wC = = 0.3226
£232, 500

4
3. You can form a portfolio of two assets, A and B, whose returns have the following charac-
teristics:

Stock Expected Return Standard Deviation Correlation Between A and B

A 10% 20% 0.5


B 15% 40%

If you demand an expected return of 12%, what are the portfolio weights? What is the
portfolio’s standard deviation?

Solution: The expected return on the portfolio is calculated from:

E (Rp ) = wA E (RA ) + wB E (RB )

Since there are only two assets in the portfolio we can define wB = 1 − wA and rewrite the
expected return on the portfolio as:

E (Rp ) = wA E (RA ) + (1 − wA ) E (RB )

Plugging in the numbers for the expected returns we can solve for wA :

0.12 = wA 0.1 + (1 − wA ) 0.15

0.12 − 0.15
wA = = 0.6
0.1 − 0.15

Therefore to obtain an expected return of 12% on the portfolio hold 60% in Stock A and
40% in Stock B.

5
The portfolio’s variance is calculated from:

σR2 p = wA
2 2 2 2
σ RA + w B σRB + 2wA wB σRA σRB ρRA ,RB

Plugging in the numbers:

σR2 p = (0.6)2 (0.2)2 + (0.4)2 (0.4)2 + 2 (0.4) (0.6) (0.2) (0.4) (0.5) = 0.0592

Therefore the portfolio’s standard deviation is:


σRp = 0.0592 = 0.2433

You might also like